0 Daumen
822 Aufrufe

Aufgabe:

(Bestimmen Sie (ohne Differation) jenes lineare Polynom p(x), welches

$$\int \limits_{0}^{1} (p(x)-x^3)^2 dx$$
minimiert.

Problem/Ansatz:

Ich hätte nun mal integriert, die Grenzen eingesetzt und mir dann p(x) ausgerechnet. Mir ist nur nicht klar, wie ich das ganze minimieren soll.

Oder geht das ganz anders! Wir behandeln zur zeit in der Linearen Algebra das Vektor-Produkt - hat das vielleicht irgendwas damit zu tun?

Danke für die Hilfe!

Avatar von

2 Antworten

0 Daumen
 
Beste Antwort

Wenn es um eine Lösung im Rahmen der Skalarprodukt-Theorie geht, wie im Kommentar angedeutet:

Wir haben also die stetige Funktion \(f(x):=x^3\). Wir haben den linearen Unterraum U der Polynome vom Höchstgrad 1 und suchen ein \(p \in U\) mit

$$(p-f)\bullet (p-f) \leq (q-f)\bullet (q-f) \quad \forall q \in U$$

Kriterium ist:

$$p-f \perp q \quad \forall q \in U$$

In diesem Fall wird U von 2 Elementen aufgespannt, zum Beispiel: \(g(x):=x, h(x):=1\) Dementsprechend hat das gesuchte p eine Darstellung \(p=ag+bh\). Die o.g. Orthogonalitätsbedingung ist dann:

$$(ag+bh-f)\bullet g=0 \text{  und } (ag+bh-f)\bullet h=0$$

Dies ist nach Einsetzen und Ausrechnen ein lineares Gleichungssystem für a und b mit den bereits angegebenen Lösungen a=0.9 und b=-0.2

Gruß Mathhilf

Avatar von 13 k

Suuuuper, vielen Dank für die detailierte Hilfe, das hilft mir jetzt sehr weiter

nochmal eine blöde Frage bitte, was genau setz ich da jetzt ein?

Für g setzt Du x ein, für h die Konstante 1 und für f x^3

super, vielen Dank, dann war meine Vermutung doch richtig

dann komm ich auf das, denke das mal ist kein mal oder?

$$I:(ax+b-x^{3})*x=0\\II:(ax+b-x^{3})*1=0\\\\I:ax^{2}+bx-x^{4}=0\\II:ax+b-x^{3}=0\\\\einsetzen\\ax^2+x^4-ax^2-x^4=0$$

Nein das \(\bullet\) soll für Skalarprodukt stehen (wie im Kommentar zuvor verwendet). Also \(p \bullet f=\int_0^1 p(x)f(x)dx\)

OK,  vielen Dank für die ausführliche Hilfe,  leider ging ich immer noch an dem wie ich da genau das Skalarprodukt berechne damit ich auf das Gleichungssystem komm,  irgenwie fehltir da leider das Verständnis wie ichs umsetzen kann

@mathhilf, bitte entschuldige, aber darf ich dich fragen wie man genau $$ (ax+b-x^3)\bullet x $$ berechnet? da steh ich noch an und google hat mir da leider auch nicht weiter geholfen...Vielen Dank für deine Geduld

$$\int_0^1(ax+b-x^3)x\;dx$$

vielen, vielen dank

0 Daumen
Ich hätte nun mal integriert, die Grenzen eingesetzt und mir dann p(x) ausgerechnet. Mir ist nur nicht klar, wie ich das ganze minimieren soll

Ich würde eher p(x) als ax+b ansetzen, das Integral damit ausrechnen und dann schauen, ob man ohne Differenziation Werte a und b so findet, dass das ganze minimal wird.

Auf alle Fälle (mit deinem oder mit meinem Weg): Anfangen!

Bevor du ans Minimieren denkst, brauchst du den konkreten Term des Integrals.

Avatar von 53 k 🚀

Ich hätt mal raus \( \frac{1}{7} \)-\( \frac{p(x)}{2} \)+\(p(x)^{2} \)

Das wird minimal, wenn \( p(x)^{2}-\frac{p(x)}{2}  \) bzw.   \(p(x)( p(x)-\frac{1}{2} ) \) minimal wird.

Das Produkt der beiden linearen Terme p(x) und \(( p(x)-\frac{1}{2} ) \) ist ein quadratischer Term. Der Graph davon ist eine Parabel. Diese hat ein lokales Minimum, wenn es sich um eine nach oben geöffnete Parabel handelt. Das Minimum liegt genau in der Mitte zwischen den beiden Nullstellen der quadratischen Funktion.

Für unterschiedliche Terme p(x) gibt es natürlich unterschiedliche Minimumstellen und dort auch unterschiedliche Minimalwerte. Nach meinem Verständnis musst du den minimalsten aller Minimalwerte finden.

Das heißt eigentlich muss ich mir nur die Scheitelpunktform berechnen, oder? gilt das auch für deine Version. Da hätte ich als Ergebnis

\( \frac{a^{2}}{3} \)+\(a( b-\frac{2}{5} \))+\( b^{2} \) -\( \frac{b}{2} \)+\( \frac{1}{7} \)


schaut etwas komplizierter hier aus ^^

Ok mit deinem Hinweis, den Minimalsten der Minimalsten hab ich eben das Problem, dass mir nicht klar ist wie ich das angehen soll nachdem ich ja auch nicht differenzieren darf

Ich weiß nicht, ob ich völlig auf den Holzweg bin.

Aber: Wo ist \(p(x)( p(x)-\frac{1}{2} ) \) gleich 0?

Offensichtlich da, wo p(x)=0 ist. Und auch da, wo p(x)=1/2 ist.

Wo ist die Mitte dieser beiden Stellen? Da p(x) eine lineare Funktion ist, ist das dort, wo p den Wert in der Mitte zwischen 0 und 1/2 annimmt → also da, wo p(x)=1/4 gilt.

Damit ist letztendlich egal, wo das konkret ist: Wichtig ist, dass p(x)=1/4 gilt und als solches in deinen ermittelten Term eingesetzt werden kann.

Heißt das dann ich muss a und b gar nicht bestimmen?


wenn ich die 1/4 einsetzte komme ich auf 9/112 was ist das dann genau? mein p(x)?

Hier soll doch nur die orthogonale Projektion von x^3 auf den Raum der Polynome vom Grad ≤1 bestimmt werden. Ich erhalte 0,9x - 0,2.

wie ist denn da die Vorgangsweise um dieses Ergebnis (0,9x-0,2) zu erhalten?

Man arbeitet im Vektorraum der Polynome (hier : vom Grad ≤ 3) mit dem Skalarprodukt
f•g = 01 f(x)·g(x) dx

ok, das sagt mir zwar was allerdings steh ich da etwas an, hast du zufällig ein beispiel welches ich mir anschauen könnte? oder könnest mir kurz sagen wie man das dann am besten macht und anfängt?

vielen Dank für den link und deine Hilfe, hätte ich zwei Vektoren würde ich mir leichter tun, mein Problem ist, dass ich nicht weiß was ich da mit f(x) und g(x) anfangen soll oder wie ich das ummünze

vor allem weil ich ja auch kein f(x)*g(x) habe bei meinem Integral oder?

hätte ich zwei Vektoren würde ich mir leichter tun

f und g sind Vektoren.
Soviel von mir, falls weitere Erklärungen nötig sind, wird die jemand anders geben müssen.

ok, trotzdem danke, vielleicht kann mir ja noch jemand helfen und sagen wie ich das angehen kann - welche schritte da notwendig sind damit ich auf 0,9x-0,2 komme


das f(x) und g(x) vektoren sind war mir schon klar, wei0 nur immer noch nicht wie ich das auf mein Integral ummünzen kann und die beiden bestimmen soll

Ein anderes Problem?

Stell deine Frage

Willkommen bei der Mathelounge! Stell deine Frage einfach und kostenlos

x
Made by a lovely community